Symmetrien in QM und QFT --- Operatortransformationsgesetze

In der Quantenmechanik implementieren wir Transformationen durch Operatoren U die den Staat abbilden | ψ zum Staat U | ψ . Alternativ könnten wir die Aktion von übertragen U auf unsere Operatoren:

Ö U Ö U
Diese Operatoren U sollte eine Repräsentation der interessierenden Transformationsgruppe darstellen. Die Frage, die wir uns dann stellen, lautet: Welche Darstellung sollen wir wählen? Ich habe gelernt, dass wir die Matrizen festlegen U das wir wünschen, indem wir verlangen, dass sich der Positionsoperator oder der Positionseigenzustand auf die geeignete geometrische Weise transformiert: Wenn R eine Rotationsmatrix in 3D ist, dann wünschen wir uns das
U ( R ) X U ( R ) = R X

Ich habe mehrere Fragen zu diesem Verfahren:

1) Das wird oft behauptet U ist einheitlich, da 'Wahrscheinlichkeiten erhalten bleiben müssen'. Aber aus meinen Studien geht hervor, dass Operatoren, die Lorentz-Boosts darstellen, nicht einheitlich sind. Wie lassen sich diese beiden Tatsachen vereinbaren?

2) In einem ähnlichen Zusammenhang sagen wir, dass eine Transformation eine Symmetrie des Systems ist , wenn der Hamiltonoperator unter der Transformation erhalten bleibt. Das ergibt für mich Sinn, da es der Hamiltonian ist, der das System definiert; Wenn der Hamilton-Operator unverändert bleibt, sind die alten Lösungen der Schrödinger-Gleichung immer noch Lösungen. Es ist jedoch klar, dass jeder Transformationsoperator, der einen Galilei-Boost (in der nicht-relativistischen QM) oder einen Lorentz-Boost (in der relativistischen QFT) implementiert, den Hamilton-Operator nicht unverändert lassen wird. Das bedeutet jedoch nicht, dass Boosts keine Symmetrien unseres Systems sind. Was ist denn hier los? Warum genau ist U H U = H die Bedingung für U eine 'Symmetrie' sein?

3) Verlangen, dass die Betreiber U die Eigenschaft erfüllen (Übersetzung für Konkretheit wählen)

U X U = X + A
sagt uns nicht sofort, wie sich die Transformation auf andere Operatoren auswirkt, wie z. B. Impuls- oder Drehimpulsoperatoren. Für den Fall einer Übersetzung von A , wir glauben, dass
U = exp ( ich A P )
die obige Gleichung erfüllt (infinitesimal arbeiten). Das sagt uns das dann U P U = P . Das wollen wir – Translationen im Raum haben keinen Einfluss auf den Impuls. Aber wir haben es unseren Mitarbeitern nicht gesagt U dass sie diese Eigenschaft erfüllen sollten, also scheint die Tatsache, dass sie es tun, eher ein Zufall zu sein. Das heißt, es ist nur erforderlich, den Positionsoperator mit zu konjugieren U ( R ) dreht, scheint es jeden Vektoroperator zu zwingen, unter Konjugation mit gedreht zu werden U ( R ) . Liegt dem etwas Grundsätzliches zugrunde?

4) In QFT wollen wir Lorentz-Boosts mit Operatoren implementieren

S = exp ( ich ω μ v M μ v 2 )
Wo δ v μ + ω μ v ist die infinitesimale Lorentz-Transformation, die wir darstellen. Jetzt für T die eine andere, nicht notwendigerweise infinitesimale Lorentz-Transformation darstellt Λ , wir haben
T 1 S T = T 1 ( 1 ich ω μ v M μ v 2 ) T
Aber die Bedingung, dass die Matrizen S Und T eine Darstellung der Lorentz-Gruppe darstellen muss, impliziert, dass die Kombination auf der linken Seite gleich sein sollte
exp ( ich Ω μ v M μ v 2 )
Wo Ω μ v definiert die zusammengesetzte Transformation:
δ v μ + Ω μ v ( Λ 1 ) μ ρ ( δ σ ρ + ω ρ σ ) Λ σ v
Der Vergleich dieser Gleichungen ergibt dann
T 1 M μ v T = Λ μ ρ Λ v σ M v σ
unter Verwendung der Hebe- und Senkeigenschaften der Minkowski-Metrik. Meine Frage ähnelt der in 3) --- das Objekt M μ v ist nur eine Sammlung von 6 Operatoren, und es ist überhaupt nicht offensichtlich, dass diese Indizes Tensor-Indizes sein sollten. Und doch irgendwie --- nur indem man nach den Matrizen fragt S um eine Darstellung zu bilden --- wir haben es geschafft, das Tensortransformationsgesetz zu reproduzieren (zugegebenermaßen haben wir a T 1 hier eher als a T --- das bezieht sich auf meine erste Frage). Mit anderen Worten, wir haben die korrekte geometrische Wirkung der Lorentz-Transformation auf die Operatoren M μ v , obwohl wir nie danach gefragt haben!

Wie Sie sehen können, bin ich sehr verwirrt von der ganzen Situation. Ich entschuldige mich, wenn die obige Diskussion etwas durcheinander klingt, und ich würde mich über jede Hilfe freuen, die gegeben werden kann!

Ihre erste Frage ist ein Duplikat von Boosts are non-unitary! . Ebenfalls verwandt ist der Satz von Wigner . Ihre dritte Frage ist irgendwie unsinnig - natürlich haben wir dem Operator gesagt, dass er das Momentum unberührt lassen sollte, denn es ist das Momentum, das Raumübersetzungen erzeugt! Die vierte Frage verstehe ich auch nicht - die M μ v sind Elemente der Lie-Algebra der Lorentz-Gruppe, also gibt es die natürliche adjungierte Wirkung der Gruppe auf ihre Algebra
Zu 4): die Elemente T liegen in einer Darstellung der Lorentz-Gruppe, und M μ v liegen in der entsprechenden Darstellung der Lie-Algebra. Wir können daher die adjungierte Wirkung der Gruppe auf die Algebra betrachten, und diese sendet M μ v Zu T 1 M μ v T per Definition. Was ich nicht verstehe, ist, warum dies gleich der rechten Seite ist, die einfach eine Lorentz-Transformation ist, die auf einen Rang-2-Tensor wirkt.

Antworten (1)

Hier ist meine Antwort auf einige Ihrer Fragen – dies basiert ausschließlich auf meinem Verständnis dieser Konzepte und könnte falsch sein.

(1) Wann immer Lorentz-Transformationen eine Symmetrie irgendeines Quantensystems sind, müssen sie notwendigerweise durch unitäre lineare Transformationen auf dem Quanten-Hilbert-Raum des Systems dargestellt werden . Operatoren, die Lorentz-Boosts auf einem relativistischen Quantensystem darstellen, sind daher einheitlich, wie in einem der in den Kommentaren erwähnten Links angegeben.

(2) Die Bedingung U H U = H kann im Allgemeinen keine notwendige und hinreichende Bedingung für a sein U eine Symmetrietransformation in der relativistischen Quantenmechanik sein. Eine Möglichkeit, dies zu sehen, besteht darin, einen Ein-Teilchen-Eigenzustand eines freien Klein-Gordon-Feldes zu betrachten | P und beobachte, dass unter einer Lorentz-Transformation der Erwartungswert P | H | P (was die Energie dieses Teilchens angibt) sollte sich wie die umwandeln 0 T H Komponente des Energie-Impuls-Viervektors.

P | U ( Λ ) H U ( Λ ) | P = Λ 0 μ P | P μ | P
und nicht als unveränderlich gelassen werden U H U = H würde bedeuten. Gleichzeitig ist die Lorentz-Transformation eine Symmetrie dieses Systems, gerade weil für alle vier Impulskomponenten ähnliche Transformationsgesetze gelten – was dem Skalar entspricht P μ P μ unter einer Lorentz-Transformation invariant bleiben.

Der hier zu beachtende Punkt ist daher, dass die Anforderung, dass eine Transformation eine Symmetrie sein muss, im Allgemeinen nicht in eine Kommutierung mit dem Hamilton-Operator übersetzt wird. Dies geschieht in den Spezialfällen der zeitunabhängigen Symmetrietransformationen (zB - räumliche Translationen und Rotationen in der nichtrelativistischen QM). Das allgemeinste Kriterium ist, dass die Aktion unter der betreffenden Transformation unveränderlich (bis auf eine Konstante) ist.

(3) Beziehen Sie sich auf einen guten Text zur Quantenmechanik (z. B. Sakurai, Moderne Quantenmechanik, Kapitel 2 und 4), der Ihre Frage beantworten sollte.

(4) Das hier erhaltene Ergebnis (dass die Generatoren der Lorentz-Gruppe sich wie Tensoren transformieren sollten) ist nicht sehr überraschend. Ein elementares (wenn auch nicht sehr allgemeines) Argument ist hier das folgende - für jede Observable, die klassischerweise ein Tensor ist (wie der Impuls-4-Vektor), muss es einen entsprechenden selbstadjungierten Operator auf einem quantenmechanischen Hilbert-Raum geben. Der Erwartungswert dieses Operators für jeden Zustand sollte sich notwendigerweise wie die Tensorobservable selbst transformieren. Dies ist eine Folge des Grundparadigmas der Quantenmechanik (Erwartungswerte repräsentieren physikalisch messbare Größen). Also muss man haben

T 1 S μ v T = Λ μ ρ Λ v σ S ρ σ
ganz allgemein für jeden Tensoroperator S μ v .

PS: Wie eingangs erwähnt, basiert diese Antwort auf meinem Verständnis der Konzepte von "Symmetrie" und "Lorentz-Kovarianz" - natürlich würde ich mich über konstruktive Kritik freuen.